Estados propios del campo cuántico funcional de ondas de Schrödinger

La razón por la que tengo este problema es que estoy tratando de resolver el problema 14.4 del libro QFT de Schwartz, que me ha confundido durante mucho tiempo.

El problema es construir los estados propios de un campo cuántico. ϕ ^ ( X ) , tal que

ϕ ^ ( X ) | Φ = Φ ( X ) | Φ .
Creo que el estado propio debería ser

| Φ = mi d 3 X 1 2 ( Φ ( X ) ϕ ^ + ( X ) ) 2 | 0
dónde ϕ ^ + ( X ) es la parte de ϕ ^ ( X ) que solo incluye operadores de creación. No he encontrado ningún libro que hable sobre este problema, por lo que en realidad no estoy seguro de si este es el resultado correcto.

Y de manera similar el estado propio de π ^ ( X ) tal que

π ^ ( X ) | Π = Π ( X ) | Π
debiera ser
| Π = mi d 3 X 1 2 ( Π ( X ) + ϕ ^ + ( X ) ) 2 | 0 .
La Ecuación 14.21 y la Ecuación 14.22 del libro de Schwartz son
Π | Φ = Exp [ i d 3 X Π ( X ) Φ ( X ) ]
Φ | Φ = D Π Φ | Π Π | Φ = D Π Exp ( i d 3 X Π ( X ) [ Φ ( X ) Φ ( X ) ] )
Ahora mi pregunta es ¿cómo verifico estas relaciones? He perdido mucho tiempo en este problema sin ningún éxito.

¿Y por qué ningún libro habla de este problema? Me refiero al estado propio de un campo cuántico. ¿No es este tipo de cosas muy fundamentales para QFT?

encontrará algunas referencias útiles en physics.stackexchange.com/a/231309/84967 Además, consulte el libro de Weinberg sobre QFT, Vol I., capítulo 9.
@AccidentalFourierTransform Weinberg solo asume que tales estados existen y luego no habló sobre qué son o cómo construirlos.
El problema es formalmente idéntico a su 312004 , y ciertamente es más fácil de explicar en el oscilador tonto, en lugar de peroratar en los campos cuánticos. Los largueros y los encendedores exprimidos lo tienen para el desayuno. Demuestra que reconoces el SU(1,1) involucrado.
@Nahc: Referencias: lss.fnal.gov/archive/other/nr-1-1995.pdf , Greiner - Field Quantization - página no. 365.
puedes interpretar este idioma de una manera "intuitiva" si tienes este "diccionario": physics.stackexchange.com/a/694149/226902

Respuestas (4)

Entonces, ésta es la cuestión. Publico aquí mi respuesta a 312004 , ya que esa está cerrada; al mismo tiempo que insisto, este aquí tiene su respuesta en un oscilador: como es habitual en QFT, la infinidad de osciladores es solo una cortina de humo para probar la comprensión del estudiante de la segunda notación de cuantificación, solo la parte b) de su problema de texto. En un mundo ideal, esta pregunta de FT anterior debería fusionarse con la cerrada, partes a) yc).

Y no, los estados propios de campo no son una parte integral de la caja de herramientas de QFT, dado el resultado esperado de QFT, al menos en física de partículas, el enfoque de su texto. Sin embargo, la teoría del funcional de ondas de Schroedinger ( Jackiw, 1988 y Lüscher, 1985 ) sigue flotando.

(Los estados coherentes se usan más comúnmente, por ejemplo, el texto QFT de Itzykson y Zuber, ecuación (3-65) y siguientes).



Esencialmente respondiendo 312004 :

Este es el anverso/completado de la pregunta gemela 292899 . Depende de un punto que no es poco común en los estados apretados y las discusiones de cuerdas. Me apresuraré a encontrar la respuesta ajustando adecuadamente las técnicas estándar, por ejemplo, de Fischer, Nieto y Sandberg, 1984 , y dejándole a usted arreglar las normalizaciones numéricas a su entera satisfacción.

Recuerdo [ a , a ] = 1 .

| X norte ( X )   Exp ( ( a X 2 ) 2 / 2 ) | 0   X ^   | X = X | X , | pags norte ( pags )   Exp ( ( a + i pags 2 ) 2 / 2 ) | 0   pags ^   | pags = pags | pags   .
Puedes intentar arreglar norte ( X ) = mi X 2 / 2 / π 1 / 4 de 0 | X , el estado fundamental de Schrödinger del oscilador. (¡Sí, de hecho, n es la inversa de la Gaussiana!)

Su asignación es en orden normal

X | pags = norte ( X ) norte ( pags ) 0 | mi ( a X 2 ) 2 / 2 mi ( a + i pags 2 ) 2 / 2 | 0   .
Es decir, conmutar todo a s a la derecha, donde aniquilan el vacío, y así desaparecen ellos mismos, dejando las x 's, etc, y lo mismo para los operadores de creación sobre el vacío a la izquierda, depositando sus p s.

En realidad, dada esta tarea, es más fácil hacer un pedido normal, en cambio, los nuevos osciladores

b a X 2 , b a + i pags 2 , [ b , b ] = 1.
De este modo,
b | 0 = X 2 | 0 , 0 | b = 0 | i pags 2 ,
de modo que la expresión de orden normal generará <0|0> veces una función de x cancelando n(x) , lo mismo para p , y el mi i X pags buscado.

La respuesta (derivada de movimientos estándar en el Apéndice a continuación) es

mi b 2 / 2 mi b 2 / 2 = mi b 2 / 4 mi en 2 ( 1 / 2 + b b ) mi b 2 / 4 = mi b 2 / 4 ( k = 0 ( 1 / 2 ) k k ! b k b k ) mi b 2 / 4 / 2 .
En consecuencia, la vev del operador anterior es
pags | X = mi pags 2 / 2 mi i X pags mi X 2 / 2 norte ( X ) norte ( pags ) / 2 π = mi i X pags / 2 π .

Para probar la ortogonalidad, por ejemplo, de los estados de posición, bien, inserte un conjunto completo de estados propios de momento en X | y , y haz la integral dp de las dos ondas planas que acabas de encontrar, dándote una   d ( X y ) .


Apéndice . Los lemas cruciales en la derivación son:

  • Definición

    L b 2 / 2 , L + b 2 / 2 , L 0 1 2 ( 1 / 2 + b b ) ,
    tenga en cuenta que se cierran en el álgebra de SU (1,1) (subyacente al álgebra de Virasoro),
    [ L + , L ] = 2 L 0 , [ L 0 , L ± ] = ± L ± .

  • El punto clave: una identidad de elemento de grupo (producto de exponenciales de generadores) es válida para todas las representaciones, pero, a la inversa, una identidad de tales elementos de grupo en una representación fiel, como el doblete (las matrices de Pauli), no puede ser válida si el genérico el abstracto no lo hace. (Esto no es trivial: requiere el teorema exponencial de Poincaré, en el sentido de que la expresión CBH que se encuentra en el exponente está completamente en el álgebra de Lie y, por lo tanto, la representación es irrelevante). Entonces, en esta representación más simple,

    L + = ( 0 1 0 0 ) , L = ( 0 0 1 0 ) , 2 L 0 = ( 1 0 0 1 ) .
    Entonces es evidente que mi L mi L + = mi L + / 2 mi en 2 2 L 0 mi L / 2 , ya que
    ( 1 0 1 1 ) ( 1 1 0 1 ) = ( 1 1 / 2 0 1 ) ( 1 / 2 0 0 2 ) ( 1 0 1 / 2 1 ) .

  • Sin embargo, el elemento del grupo medio, mi en 2   ( 1 / 2 + b b ) , todavía no está ordenado normalmente, pero es fácil llegar allí, dada la identidad

    mi C b b = k = 0 ( mi C 1 ) k k ! b k b k .
    (Simplemente escriba las primeras cuatro potencias de c en la expansión ascendente del teorema de Wick en b s de orden normal. Esto se atribuye a Wilcox (1967) JMP 8 962-982 y, en última instancia, a McCoy, 1932, pero es casi manifiesto. El propagador del oscilador.)

La forma más corta de verlo es explotando el isomorfismo algebraico b = X ,     b = ; de modo que el lado izquierdo que actúa sobre f(x) solo está escalando, Exp ( C X )   F ( X ) = F ( mi C X ) ; mientras que la de la derecha es la serie de Taylor alrededor de x , desplazada por X ( mi C 1 ) , y por lo tanto F ( X ) F ( X + X ( mi C 1 ) ) = F ( mi C X ) .


Volviendo a la teoría de campos: todo lo que necesita hacer ahora es considerar una infinidad de osciladores, de donde, vagamente, X ^ ϕ ^ ( X ) , pags ^ π ^ ( X ) , construya los estados propios como en 292899 y generalice lo anterior para Π | Φ , mutatis mutandis . Como consecuencia directa, observe que el estado invariante traslacionalmente en el núcleo del impulso es

| Π = 0 = mi d 3 X   ϕ ^ + ( X ) 2 / 2 | 0 .

Intenté hacer el problema mutatis mutantdis . Creo que no es tan trivial como suena. los 1 / mi pags en la medida de integración invariante de lorentz conduce a problemas con hacer el problema de manera análoga, y la única solución para el estado propio del campo tenía un producto de operadores en el exponente, lo que me dejó sin idea de cómo probar la relación de ortogonalidad.
Sí, las normalizaciones y las relaciones de conmutación alteradas de los modos son parte de los mutandis , por supuesto. ¡Puede rastrear ti en su texto para asegurarse de cómo muta! A partir de los modos separados, un producto de exponenciales produce un exponencial de la suma de los respectivos exponentes, ¿no?
¿Tiene una referencia para esto? No conozco ningún texto donde se haga esto y no conozco un manual de soluciones para el libro Schwartz QFT. Dividir las integrales en productos infinitos es una idea interesante que aún no he probado, y lo haré si tengo un momento para hacerlo.
Lo siento, no. El libro de Lowell Brown es de muy buen gusto...
Espero que esto no sea tarea. Has revisado este y sus enlaces, ¿no?
Pude verificar el estado propio en ese momento pero no la relación de conmutación. ¿Sería útil escribir la conjetura que usé para el estado propio?
También encontré el estado propio y se ve (hasta las convenciones) como el de physics.stackexchange.com/questions/456985/… . Ahora estoy tratando de probar 14.21 en schwartz y no estoy seguro si los estados que encontré obedecen a esta relación y me interesaría si la cumplen. ¿Quiere decir por relación de conmutación dicho 14.21? (perdón, borré mi comentario porque lo encontré antes de ver sus respuestas)
@lomby Fuiste tú a quien envié a ese enlace en mi comentario 2 artículos arriba aquí... ¡Parece parte de otra conversación ahora! Schwartz sec 2.57 y 14.2.3 cubren el paseo marítimo. Lea y verifique su nota al pie.
@CosmasZachos Sí, lo siento, borré la publicación. He leído las cosas en Schwartz pero todavía estoy atascado. Eche un vistazo a mi comentario en physics.stackexchange.com/questions/456985/…
@CosmasZachos Para tu última ecuación, implicas que a / 2 d 3 X ( ϕ + ( X ) ) 2 / 2 . En el caso más general (momento distinto de cero), tenemos el total ( a + i 2 pags ) 2 (según su respuesta), entonces su generalización no debería tener una integral doble 1 2 d 3 X d 3 y ( ϕ + ( X ) + . . ) ( ϕ + ( y ) + . . ) ? (donde supongo que el . . es algo proporcional a i Π ( X ) )
@TheQuantumMan No, no veo cómo te imaginas, quiero decir que para la extensión del estado de vacío traduccionalmente invariable de Dirac...
@CosmasZachos En la segunda ecuación que escribiste, | pags = 0 mi X pags ( ( a ) 2 / 2 ) | 0 . En la ecuación final que escribiste | Π = 0 = mi X pags ( d 3 X   ϕ + 2 ( X ) / 2 ) | 0 .
Sí, integral simple, entonces producto de exponencial de bilineales indexados por x o momento.

Mi solución se refiere a las respuestas de Cosmas Zachos en esta pregunta y 292899 . Adoptaré nociones del libro QFT de Swhartz. Además, terminar el problema 14.3 será útil para resolver este problema.

a)

| X = 1 π 1 / 4 Exp ( X 2 / 2 ) Exp ( ( a X 2 ) 2 / 2 ) | 0 | pags = 1 π 1 / 4 Exp ( pags 2 / 2 ) Exp ( ( a + i pags 2 ) 2 / 2 ) | 0
b) Introducción de la convención para describir el "producto interno" de los operadores de campo:
< A ^ ( X ) , B ^ ( y ) > F ( X , y ) := d 3 X d 3 y A ^ ( X ) B ^ ( y ) F ( X , y )
dónde F es una función Entonces los estados propios de ϕ ^ ( X ) ,   π ^ ( X ) son:
| Φ = norte Exp ( < Φ , Φ > mi / 2 ) Exp ( < ϕ ^ + Φ , ϕ ^ + Φ > mi ) | 0 | Π = norte Exp ( < Π , Π > mi 1 / 2 ) Exp ( < π ^ + Π , π ^ + Π > mi 1 ) | 0
dónde norte es un número de normalización, y
mi ( X , y ) = d 3 pags ( 2 π ) 3 ω pags mi i pags ( X y ) mi 1 ( X , y ) = d 3 pags ( 2 π ) 3 1 ω pags mi i pags ( X y )
Para verificar estas respuestas, primero calcule
[ ϕ ^ ( z ) , F ( < ϕ ^ + Φ , ϕ ^ + Φ > mi ) ] = d F d < ϕ ^ + Φ , ϕ ^ + Φ > mi ( ϕ ^ + ( z ) Φ ( z ) )
Después
ϕ ^ ( z ) | Φ = ϕ ^ + ( z ) | Φ + ϕ ^ ( z ) | Φ = ϕ ^ + ( z ) | Φ + [ ϕ ^ ( z ) , norte Exp ( < Φ , Φ > mi / 2 ) Exp ( < ϕ ^ + Φ , ϕ ^ + Φ > mi ) ] | 0 = ϕ ^ + ( z ) | Φ norte Exp ( < Φ , Φ > mi / 2 ) Exp ( < ϕ ^ + Φ , ϕ ^ + Φ > mi ) ( ϕ ^ + ( z ) Φ ( z ) ) | Φ = Φ ( z ) | Φ
Otro estado propio se puede verificar de manera similar.

C)

Definir argumento de exponencial como

L ^ + :=< ϕ ^ Φ , ϕ ^ Φ > mi L ^ :=< π ^ + Π , π ^ + Π > mi 1
calcular su conmutador
[ L ^ + , L ^ ] = i d 3 X { ( ϕ ^ Φ ) , ( π ^ + Π ) } ( X ) =: 2 L ^ 0
NOTA { , } es anti-conmutador. Después
[ L ^ 0 , L ^ ± ] = ± L ^ ±
entonces obtenemos el álgebra de Lie s tu ( 2 ) = s pags a norte C { L ^ 0 , L ^ + , L ^ } .

Según la respuesta anterior de Cosmas Zachos, tenemos

mi L ^ + mi L ^ = mi L ^ / 2 mi 2 yo norte 2 L ^ 0 mi L ^ + / 2 Exp [ 2 i λ < π ^ + Φ , ϕ ^ Φ > 1 ] = norte { Exp [ 2 i ( mi λ 1 ) < π ^ + Π , ϕ ^ Φ > 1 ] }
dónde norte es un operador de pedidos regular para organizar ( π ^ + Π ) de izquierda a ( ϕ ^ Φ ) .

Finalmente,

Φ | Π = norte 2 Exp ( < Φ , Φ > mi / 2 ) Exp ( < Π , Π > mi 1 / 2 ) 0 | mi L ^ + mi L ^ | 0 = norte 2 0 | mi 2 yo norte 2 L ^ 0 | 0 = norte 2 0 | Exp { yo norte 2 i d 3 X [ i 2 d 3 ( 0 ) + 2 < π ^ + Φ , ϕ ^ Φ > 1 ] | 0 = norte 2 Exp ( 1 norte 2 2 d 3 X d 3 ( 0 ) ) 0 | norte { Exp [ 2 i ( mi yo norte ( 1 / 2 ) 1 ) < π ^ + Π , ϕ ^ Φ > 1 ] } | 0 = Exp [ i d 3 X Π ( X ) Φ ( X ) ]
donde nos ponemos norte = Exp ( 1 norte 2 4 d 3 X d 3 ( 0 ) ) .

También probé el ejercicio 14.4 del libro de Schwartz. (Uso las mismas convenciones y notaciones que en el libro) Mi conjetura para el estado propio dice

| Φ = Exp [ d 3 X d 3 y ( Φ ( X ) ϕ ^ + ( X ) ) mi ( X , y ) ( Φ ( y ) ϕ ^ + ( y ) ) ] | 0
dónde
mi ( X , y ) = d 3 pags ( 2 π ) 3 mi i pags ( X y ) ω pags .
Pude haber cometido algún error de cálculo pero creo que el Ansatz debería ser correcto. Del mismo modo, para los estados propios de impulso obtengo
| Π = Exp [ d 3 X d 3 y ( Π ( X ) π ^ + ( X ) ) mi 1 ( X , y ) ( Π ( y ) π ^ + ( y ) ) ] | 0
dónde
mi 1 ( X , y ) = d 3 pags ( 2 π ) 3 mi i pags ( X y ) 1 ω pags
es justo lo contrario de mi . Sin embargo, todavía tengo problemas para verificar
Φ | Π = Exp [ i d 3 X Φ ( X ) Π ( X ) ] .
(Esto y la relación de completitud implican inmediatamente la ortogonalidad). Quiero usar https://en.wikipedia.org/wiki/Baker%E2%80%93Campbell%E2%80%93Hausdorff_formula . si defino A como el exponente punzante en | Φ y B como exponente en | Π . consigue eso
[ A , B ] = 2 i d 3 X [ ( Φ ϕ ^ ) ( Π π ^ + ) + ( Π π ^ + ) ( Φ ϕ ^ ) ] .
Darse cuenta de
0 | [ A , B ] | 0 = 4 i d 3 X Π ( X ) Φ ( X ) +  constante infinita
así que ya casi llego (factores mi A y mi B se convierten en escalares cuando se intercalan entre estados de vacío y pueden extraerse y absorberse mediante la normalización). Sin embargo, en la fórmula de Baker-Campbell-Hausdorff aparecen más términos en el exponente. Por lo que puedo ver, estos términos (después de estar intercalados) pueden ser absorbidos por la normalización o son proporcionales a [ A , B ] (ya que [ A , [ A , B ] ] A ,   [ B , [ A , B ] ] B ), por lo que solo me deja con una descripción de un prefactor desconocido en el exponencial (tal vez después de la suma de la serie infinita, el prefactor se convierte en 1 como se dice?).
También es probable que haya cometido un error y agradecería si alguien pudiera señalarlo y/o mostrarme la solución.

Hola, ¿por qué incluiste el mi en tu ansatz?
@TheQuantumMan Ha pasado algún tiempo desde que hice este cálculo. Si no recuerdo mal, también se puede mi ( X , y ) = d ( X y ) y sigue siendo una función propia. Pero encontré esta forma más conveniente para mostrar las otras relaciones (¿si es que las cumplen?). También recuerdo que vi las ecuaciones (14.65) y (14.66) en Schwartz, que sugieren más la forma que tengo. También recuerdo que estaba bastante insatisfecho con mi solución, así que si conoces una solución, házmelo saber :)
¡A partir de ahora, tampoco estoy satisfecho con mi propia solución! Si tengo algo que valga la pena compartir, dejaré un comentario :)
He verificado todo. La única diferencia en mi cálculo es [ A , B ] = i en vez de [ A , B ] = 2 i . POR CIERTO, mi ( X , y ) = d ( X y ) no funcionaría, porque necesitamos cancelar términos como ω pags / 2 o 1 / 2 ω pags en π ^ o ϕ ^ .

El problema esencialmente se reduce a la conmutación de operadores complicados. Si un estado está representado por

| ψ = Exp ( F ( a ^ , a ^ ) ) | vacaciones k ^ | vacaciones ,
dónde F ( a ^ , a ^ ) es alguna función de los operadores de escalera, entonces el producto interno genérico es de la forma
ψ 1 | ψ 2 = vacaciones | k ^ 1 k ^ 2 | vacaciones .
Para resolver el problema se necesita obtener el producto. k ^ 1 k ^ 2 en orden normal. De ahí la complicada conmutación.

Existe un procedimiento general para calcular conmutaciones complicadas de operadores, siempre que los operadores puedan representarse en forma exponencial con argumentos que son la mayoría de segundo orden en los operadores de escalera. El problema en la situación actual se puede expresar como

Exp ( F 1 ( a ^ , a ^ ) ) Exp ( F 2 ( a ^ , a ^ ) ) = A Exp ( gramo 1 a ^ 2 ) Exp ( gramo 2 a ^ a ^ ) Exp ( gramo 3 a ^ 2 ) ,
dónde A , gramo 1 , gramo 2 , y gramo 3 son las incógnitas a resolver. Tenga en cuenta que el segundo operador en el lado derecho con los operadores numéricos no está en el orden normal, pero como el vacío es un estado propio de los operadores numéricos, esto no es un problema.

El procedimiento general ahora es insertar una variable ficticia en los argumentos de los exponentes y hacer todas las funciones desconocidas de esta variable ficticia:

Exp ( t F 1 ( a ^ , a ^ ) ) Exp ( t F 2 ( a ^ , a ^ ) ) = A ( t ) Exp ( gramo 1 ( t ) a ^ 2 ) Exp ( gramo 2 ( t ) a ^ a ^ ) Exp ( gramo 3 ( t ) a ^ 2 ) ,
dónde t es la variable ficticia. Luego calcule la derivada de la ecuación con respecto a t . A continuación, elimine la mayor cantidad posible de operadores aplicando los respectivos operadores adjuntos. Entonces usas la identidad para Exp ( X ^ ) Y ^ Exp ( X ^ ) para simplificar la expresión. El resultado se puede separar en varias ecuaciones diferenciales que se pueden resolver para obtener soluciones para las incógnitas.

He resuelto este problema en su totalidad para los estados propios de los operadores de cuadratura en un artículo: Phys. Rev. A 98, 043841 (2018) - arXiv:1810.04396. Ver también la Fe de erratas: Phys. Rev.A 101, 019903(E) (2020). El análisis proporciona expresiones con las que se puede obtener el resultado para todos los productos internos requeridos.